Mr. Rodriguez is preparing photos for an international client. The client has requested a photo that is 20 cm by 15 cm. Mr. Rodriguez knows that the formula c = 2.54n can be used to convert n inches to c centimeters. Which formula can he use to convert centimeters to inches?

Mr. Rodriguez Is Preparing Photos For An International Client. The Client Has Requested A Photo That

Answers

Answer 1
Answer: [tex]n\text{ = }\frac{c}{2.54\text{ }}\text{ \lparen option B\rparen}[/tex]

Explanation:

Given:

The formula to convert from inches to centimeters is c = 2.54n

To find:

The formula that can be used to convert from centimeters to inches

To determine the formula, we need to make n the subject of formula:

[tex]\begin{gathered} \text{c = 2.54n} \\ where\text{ c = value in cm} \\ n\text{ = value in inches} \end{gathered}[/tex][tex]\begin{gathered} To\text{ make n, the subject of formula, we will divide both sides by 2.54:} \\ \frac{c}{2.54}=\text{ }\frac{2.54n}{2.54} \\ n\text{ = }\frac{c}{2.54} \\ This\text{ means when we have a value in cm and substitute, the answer will be in inches} \end{gathered}[/tex][tex]n\text{ = }\frac{c}{2.54\text{ }}\text{ \lparen option B\rparen}[/tex]


Related Questions

The number of skateboards that can be produced by a company can be represented by the function f(h) = 325h, where h is the number of hours. The total manufacturing cost for b skateboards is represented by the function g(b) = 0.008b2 + 8b + 100. Which function shows the total manufacturing cost of skateboards as a function of the number of hours? g(f(h)) = 325h2 + 80h + 100 g(f(h)) = 3425h + 100 g(f(h)) = 845h2 + 2,600h + 100 g(f(h)) = 2.6h2 + 2,600h + 100

Answers

The function which shows the total manufacturing cost of skateboards as a function of the number of hours is; g(f(h)) = 845h2 + 2,600h + 100.

Which function shows the manufacturing cost as a function of number of hours?

It follows from the task content that the function which shows the manufacturing cost as a function of the number of hours be determined.

Since, the number of skateboards is given in terms of hours as; f(h) = 325h and;

The manufacturing cost, g is given in terms of the number of skateboards, b manufactured;

The function instance which represents the manufacturing cost as a function of hours is; g(f(h)).

Therefore, we have; g(f(h)) = 0.008(325h)² + 8(325h) + 100.

Hence, the correct function is; g(f(h)) = 845h2 + 2,600h + 100.

Read more on function instances;

https://brainly.com/question/28839856

#SPJ1

Assume the random variable X has a binomial distribution with the given probability of obtaining a success. Find the following probability, given the number of trials andthe probability of obtaining a success. Round your answer to four decimal places.P(X= 15), n = 18, p = 0.8TablesKeynad

Answers

Recall that the probability of a binomial distribution is given by

[tex]P(X=x)=^^nC_r\cdot p^x\cdot(1-p)^{n-x}[/tex]

Where n is the number of trials, p is the probability of success, and x is the variable of interest.

nCr is the number of combinations.

For the given case, we have

n = 18

p = 0.8

x = 15

Let us find the probability P(X=15)

[tex]\begin{gathered} P(X=15)=^{18}C_{15}\cdot0.8^{15}\cdot(1-0.8)^{18-15} \\ P(X=15)=816\cdot0.8^{15}\cdot0.2^3 \\ P(X=15)=0.2297 \end{gathered}[/tex]

Therefore, the probability P(X=15) is 0.2297

I have question 3 and need to know a b and c

Answers

a) Recall that:

[tex]-1\le\cos \theta\le1.[/tex]

Therefore:

[tex]\begin{gathered} -1\le\cos (30^{\circ}\times t)\le1, \\ -12\le12\cos (30^{\circ}\times t)\le12, \\ -12+16\le12\cos (30^{\circ}\times t)+16\le12+16, \\ 4\le12\cos (30^{\circ}\times t)+16\le28. \end{gathered}[/tex]

Therefore the minimum height of the Ferris wheel above the ground is 4 meters.

b) Recall that to evaluate a function at a given value, we substitute the variable by the given value, then, evaluating the given function at t=3 we get:

[tex]12\cos (30^{\circ}\times3)+16.[/tex]

Simplifying the above result we get:

[tex]\begin{gathered} 12\cos (90^{\circ})+16, \\ 12\cdot0+16, \\ 0+16, \\ 16. \end{gathered}[/tex]

Therefore, the height of the Ferris wheel above the ground after 3 minutes is 16 meters.

(c) Let x be the time in minutes the Ferris wheel takes to complete one full rotation, then we can set the following equation:

[tex]30^{\circ}\times x=360^{\circ}.[/tex]

Therefore:

[tex]30x=360.[/tex]

Dividing the above equation by 30 we get:

[tex]\begin{gathered} \frac{30x}{30}=\frac{360}{30}, \\ x=12. \end{gathered}[/tex]

Answer:

(a) 4 meters.

(b) 16 meters.

(c) 12 minutes.

a positive integer is nice if there is a positive integer with exactly four positive divisors (including and ) such that the sum of the four divisors is equal to . how many numbers in the set are nice?

Answers

Answer:

A positive integer with exactly four positive divisors (including and ) such that the sum of the four divisors is equal to The sum of four divisors is equal to 45360.

What is an integer?

Zero, a positive natural number, or an unsigned negative integer are all examples of integers. The inverses of the equivalent positive numbers, which are additive, are the negative numbers. The boldface Z or blackboard bold "Z" is frequently used in mathematical notation to represent a collection of numbers.

Step-by-step explanation:

We know That total No. of factors

=product of (prime no′s power+1)

If N is the number of different divisors:

N=(p1​+1)⋅(p2​+1)⋅⋅⋅(pn​+1)

100= 2^2 × 5^2

=2×2×5×5= (1+1)(1+1)(4+1)(4+1)

Then the integer n=  a1^p1​​⋅a2^p2​​⋅⋅⋅⋅an^pn​​

For the smallest value: p1​=4,p2​=4,p3​=1,p4​=1

Then,

n=a1^4​×a2^4​×a3^1​×a4^1​

=24⋅34⋅51⋅71

=16⋅81⋅5⋅7

=45360

Hence, the positive integer with exactly four positive divisors (including and ) such that the sum of the four divisors is equal to The sum of four divisors is equal to 45360.

To learn more about the integers from the given link

https://brainly.com/question/929808

#SPJ4

12345678912345678900[tex]11447 \times \frac{333}{999} \times {141}^{2} - x \times y = \sqrt[255]{33} [/tex]Jardin De Ronda. updtCHECK EQUATION in QUESTION ! UPDT 2 :) `!!!z

Answers

test

some text with formatting

0. primo

,

1. secondo

,

2. terzo

• good

,

• better

,

• the best

jiohoh oj; lkippomklok

yes

pl

lkp

ok

=)

Where are the minimum and maximum values for f(x)A. min: x =2¹ 2Reset Selectionmax:z = = 0, π, 2πOB. min:z = π max:x = 0, 2OC. min:z = 0, 2π max:x = πOD. min:z = ,,max:x = 0, 3, 4,2=- 3 cos z - 2 on the interval [0, 2π]?2P

Answers

Given:

The function f(x) = 3cos(x) - 2.

Required:

What are the minimum and maximum value of function?

Explanation:

To check maximum and minimum value of function.

First derivate the original function.

After putting first derivative equal to zero, critical points can be found.

Then, do second deritvative to check points of maxima and minima.

The critical points at which second derivative greater than zero. Point will be of minima.

The critical points at which second derivative less than zero. Point will be of maxima.

So,

[tex]\begin{gathered} f(x)=3cos(x)-2 \\ \text{ First derivative} \\ f^{\prime}(x)=-3sinx \\ \text{ Put }f^{\prime}(x)=0 \\ sinx=0 \\ x=0,\pi,2\pi \end{gathered}[/tex]

Now, do second derivative test for maximum and minimum points

[tex]\begin{gathered} f^{\prime}^{\prime}(x)=-3cosx \\ \text{ At }x=0 \\ f^{\prime}^{\prime}(0)=-3\times1=-3<0 \\ \text{ At }x=\pi \\ f^{\prime}^{\prime}(\pi)=-3\times cos(\pi)=-3\times-1=3>0 \\ At\text{ }x=2\pi \\ f^{\prime}^{\prime}(2\pi)=-3\times1=-3<0 \\ \end{gathered}[/tex]

Answer:

[tex]\text{ The points }0,2\pi\text{ are points of maxima and }\pi\text{ giving minima.}[/tex]

giving the figure below, what is the measure of angle JKL

Answers

The measure of < JKL = 25+25 = 50 degrees.

Angle JOK = 360 -230 = 130 degrees , (where O is the center of the circle)

< OLK = < OJK = 90 degrees ( tangent to a circle)

< LOK = < JOK = 180 - (90+65) = 180 - 155 = 25 degrees

The solution is: < JKL = 25 +25 = 50 degrees

I'm graphing and I need to find out how mutch it costs for 4.5 inches of the construction. and the construction is $25.50 per inch

Answers

You have to determine the cost for 4.5 inches of the construction using the graph.

The height is on the y-axis, and the cost is on the x-axis.

First, locate 4.5 in the y-axis, which is the value in the midpoint between 4 and 5.

Draw a horizontal line until you intersect with the line, then draw a vertical line from the function until the x-axis:

The line crosses the x-axis at the midpoint between values 102 and 127.5 to determine the value at this point you have to average both costs:

[tex]\frac{127.5+102}{2}=\frac{229.5}{2}=114.75[/tex]

The cost of 4.5 inches of construction is $114.5

Vera cut a piece of fabric into 5 equal-length pieces. Then she cut another 3 centimeters off one piece, leaving 6 centimeters of fabric. How many centimeters long was her original piece of fabric? Write two equations with letters for the unknowns. Solve.

Answers

We have

fabric cut into 5 equal -length pieces

she cut 3 cm

l=length of one piece of the 5 equals pieces

l=6+3

l=9

the original piece of fabric is

o= length of the original fabric

o=5l

o=5(9)

o=45 cm

the original piece of fabric is 45 cm

The table displays the mean name length for seven samples of students.Sample1Mean Name Length5.47.1236.345.2566.04.976.2What can be said about the variation between the sample means?The variation between the sample means is small.The variation between the sample means is large.The variation shows that the values are far apart.The variation cannot be used to make predictions.

Answers

First option is correct.

For all the sample sizes, the sample mean is close to 6, give or take (

Sarina throws a ball up into the air, and it falls on the ground nearby. The ball's height, in feet, is modeled by the function ƒ(x) = –x2 – x + 3, where x represents time in seconds. What's the height of the ball when Sarina throws it?Question 12 options:A) 1 footB) 3 feetC) 4 feetD) 2 feet

Answers

Answer:

3 feet

Explanation:

We are told from the question that the ball's height, in feet, is modeled by the below function;

[tex]f(x)=-x^2-x+3[/tex]

where x = time in seconds

To determine the height of the ball when Sarina throws the ball, all we need to do is solve for the initial height of the ball, i.e, the height when x = 0. So we'll have;

[tex]\begin{gathered} f(0)=-(0)^2-(0)+3 \\ f(0)=3\text{ f}eet \end{gathered}[/tex]

I need help please. I don’t know what to do.Number 6

Answers

By definition, a relation is a function if each input value (x-value) has one and only one output value (y-value).

In this case, you have the following relation:

[tex]\mleft(1,5\mright)\mleft(3,1\mright)\mleft(5,0\mright)\mleft(-2,6\mright)[/tex]

Notice that each ordered pair has this form:

[tex](x,y)[/tex]

Where "x" is the input value and "y" is the output value.

You can identify that each input value has one and only output value. Therefore, you can conclude that this relation is a function.

Hence, the answer is: It is a function.

Inequality-x less than or equal to 18

Answers

Answer: x[tex]\leq[/tex]18

at a sale a desk is being sold for 24% of the regular price. the sale price is $182.40 what is the regular price

Answers

at a sale a desk is being sold for 24% of the regular price. the sale price is $182.40 what is the regular price​

we have that

24% ------> represent $182.40

so

Applying proportion

Find out the 100%

Let

x ----> the regular price

182.40/24=x/100

solve for x

x=(182.40)*(100)/24

x=$760

therefore

The regular price is $760

What is the measure of ?ХvO A. 46°42°42"38°NуvO B. 42°O C. 40°O D. 38°

Answers

The value Z is denoted as the center of the circle. Therefore, arc UV and arc XY should be the same .

[tex]undefined[/tex]

Answer: A. 42°

Step-by-step explanation:

Hope this helps :)

Points that lie on the same line are called: a) opposite rays b) coplanar and non-collinear c) non-collinear and non-coplanar d) collin ear and coplanar

Answers

Given:

Points that lie on the same line.

Opposite rays

Plllssss help Select all equations that are also equivalent to0.6 + 15b + 4= 25.6 ( choose all the ones down below the equal the top)A . 15b+4 = 25.6B .15b+4=25 C. 3(0.6+ 15b +4) = 76.8 D. 15b = 25.6E. 15b= 21

Answers

The given equation is

[tex]0.6+15b+4=25.6[/tex]

If we subtract 0.6 on each side, we get

[tex]\begin{gathered} 0.6+15b+4-0.6=25.6-0.6 \\ 15b+4=25 \end{gathered}[/tex]

Therefore, the given expression is equivalent to B.

If we multiply the given equation with 3, we get

[tex]\begin{gathered} 3\cdot(0.6+15b+4)=25.6\cdot3 \\ 3(0.6+15b+4)=76.8 \end{gathered}[/tex]

Therefore, the given expression is equivalent to C.

At last, if we subtract 0.6 and 4 on each side, we get

[tex]\begin{gathered} 0.6+15b+4-0.6-4=25.6-0.6-4 \\ 15b=21 \end{gathered}[/tex]

Therefore, the given expression is equivalent to E.

The right answers are B, C, and E.

How many fourteenths are there in 3/ 7 ?

Answers

The answer is 6.

This question is asking you to divide 3/7 by 1/14 (because that’s how you calculate how “many” of one number is in another number).

To divide fractions, we keep the first one the same, flip the second one upside down, and multiply them:

3/7 ÷ 1/14 -> 3/7 ÷ 14/1 -> 3/7 * 14/1

When you multiply 3/7 by 14, you get 42/7 = 6.

select all reasons that support one or more statements in ghe proof.

Answers

Answer:

B, C, D and E.

Explanation:

The proof and reasons for each step is given below:

Step 1:

Statement: RSTU is a parallelogram.

Reason: Given

Step 2:

Statement: RS is parallel to TU and RU is parallel to TS

Reason: (B)Definition of a parallelogram

Step 3:

Statement: ∠RSU≅∠TUS and ∠RUS≅∠TSU.

Reason: (C)Alternate Interior angles are congruent

Step 4:

Statement: SU≅US.

Reason: (E)corresponding parts of congruent triangles are congruent.

Step 5:

Statement: Triangle RSU≅Triangle TUS.

Reason: AAS Congruence Theorem

Step 6:

Statement: RS≅TU and RU≅TS

Reason: (D)Opposite sides of a parallelogram are congruent.

The reasons that support the proof are B, C, D and E.

Two methods to solve (X+3)^2=6

Answers

The solution of the given equation is [tex]-3+\sqrt{6}[/tex] and [tex]-3-\sqrt{6}[/tex].

Given equation:-

[tex](x+3)^2=6[/tex]

We have to find the value of x by solving the given equation.

We can rewrite the given equation as:-

[tex]x^2+6x+9=6\\x^2+6x+3=0[/tex]

We can solve the the quadratic equation by finding the discriminant.

[tex]x = \frac{-6+-\sqrt{6^2-4*1*3} }{2*1}[/tex]

[tex]x = \frac{-6+-\sqrt{36-12} }{2}[/tex]

[tex]x=\frac{-6+-2\sqrt{6} }{2}=-3+-\sqrt{6}[/tex]

Hence, the values of x are [tex]-3+\sqrt{6}[/tex] and [tex]-3-\sqrt{6}[/tex].

Discriminant

In arithmetic, a polynomial's discriminant is a function of the polynomial's coefficients.

Quadratic equation

The polynomial equation whose highest degree is two is called a quadratic equation or sometimes just quadratics. It is expressed in the form of:

ax² + bx + c = 0

where x is the unknown variable and a, b and c are the constant terms.

To learn more about quadratic equation, here:-

https://brainly.com/question/17177510

#SPJ1

Which of the following values have 3 significant figures? Check all that apply.A. 10.1B. 100.05C. 120D. 129

Answers

The number of significant figures in 10.1 is 3 as there are two digits before the decimal and one digit after the decimal.

The number of significant digit in 100.05 is 5 as there are 3 digits before the decimal and two digits after the decimal.

The number of significant digits in 120 is 2.

The number of significant digits in 129 is 3.

Hence, the correct answers are (A) and (D)digit

8ftÜ4ft7ft5ftA right angle is removed from a rectangle to create the shaded region shown below find the area of the shaded region be sure to include the correct unit in your answer

Answers

First, we need to find the sides of the triangle.

The base of the triangles is 8ft - 5ft = 3ft.

The height for the triangle is 7ft - 4ft = 3ft

Now, we need to find the area of the triangle:

[tex]A_t=\frac{base\cdot height}{2}[/tex]

Replacing the values:

[tex]A_t=\frac{3ft\cdot3ft}{2}[/tex]

Then

[tex]A_t=4.5ft^2^{}[/tex]

Now, we need to find the area for the rectangle:

Area for a rectangle = Length * Width

In this case:

Length = 8ft

Width = 7ft

Therefore:

[tex]A_r=8ft\cdot7ft[/tex]

Then

[tex]A_r=56[/tex]

Finally, to find the area of the shaded region we need to subtract the triangle area from the rectangle area:

[tex]A=A_r-A_t[/tex]

Therefore:

[tex]A=56ft^2-4.5ft^2[/tex][tex]A=51.5ft^2[/tex]

Hence, the area for the shaded region is 51.5 ft².

which is the new equation written in the slope-intercept form

Answers

he slope-intercept form​ is

y = mx + c

here, m = slope of the line and C is intercept on y axis.

Write a explicit formula for the given recursive formulas for each arithmetic sequence

9,15,21,27 and 7,0,-7,-14

Answers

In arithmetic progression, 9,15,21,27,33,39 is a₅ and    a₆ .

What is arithmetic progression?

A series of numbers is called a "arithmetic progression" (AP) when any two subsequent numbers have a constant difference. It also goes by the name Arithmetic Sequence.

a₁ = 9

a₂ = 15

a₃ = 21

Notice that a₂ - a₁ = 6 and a₃ - a₂ = 6

We can deduce that aₙ₊₁ = aₙ + 6

We can test this on the 4th term : a₄ should equal  21  + 6 = 27

Since this checks out we can say that the sequence is an arithmetic progression with a common difference of 6.

a₅ = 27 + 5 = 33

and

  a₆ = 33 + 6 = 39

7,0,-7,-14

find the common difference by substracting any term in the sequence from the term that comes after it.

 a₂ - a₁  = 0 - 7 = -7

 a₃ - a₂ = -7 - 0 = -7

 a₄ - a₃ = -14 - -7 = -7

the difference of the sequence is constant and equals the difference between two consecutive terms.

  d = -7

Learn more about arithmetic progression

brainly.com/question/11634518

#SPJ13

Consider the polynomial function q ( x ) = − 2 x 8 + 5 x 6 − 3 x 5 + 50

Answers

The function has an end behaviour of x → ∞, q(x) → -∞ and x → -∞, q(x) → -∞

How to determine the end behaviour of the function?

The equation of the polynomial function is given as

q(x) = -2x⁸ + 5x⁶ - 3x⁵ + 50

To determine the end behaviour of the function, we calculate

q(∞) and q(-∞)

So, we have

q(∞) = -2(∞)⁸ + 5(∞)⁶ - 3(∞)⁵ + 50

Evaluate the exponents

q(∞) = -2(∞) + 5(∞) - 3(∞) + 50

This gives

q(∞) = -∞ + ∞ - ∞ + 50

q(∞) = -∞

Also, we have

q(-∞) = -2(-∞)⁸ + 5(-∞)⁶ - 3(-∞)⁵ + 50

Evaluate the exponents

q(-∞) = -2(∞) + 5(∞) - 3(-∞) + 50

This gives

q(-∞) = -∞ + ∞ + ∞ + 50

q(-∞) = -∞

Hence, the end behaviour of the graph is x → ∞, q(x) → -∞ and x → -∞, q(x) → -∞

Read more about end behaviour at:

https://brainly.com/question/1365136

#SPJ1

Complete question

Consider the polynomial function q(x) = -2x⁸ + 5x⁶ - 3x⁵ + 50

Calculate the end behaviour

Determine the largest integer value of x in the solution of the following inequality.

Answers

Answer:

From the solution the largest possible integer value of x is;

[tex]-6[/tex]

Explanation:

Given the inequality;

[tex]-x-1\ge5[/tex]

To solve, let's add 1 to both sides of the inequality;

[tex]\begin{gathered} -x-1+1\ge5+1 \\ -x\ge6 \end{gathered}[/tex]

then let us divide both sides of the inequaty by -1.

Note: since we are dividing by a negative number the inequality sign will change.

[tex]\begin{gathered} \frac{-x}{-1}\leq\frac{6}{-1} \\ x\leq-6 \end{gathered}[/tex]

Therefore, From the solution the largest possible integer value of x is;

[tex]-6[/tex]

The total payroll for a baseball team is 2.44 × 109 dollars, and the total payroll for a football team is 2.9 × 1011 dollars. How many more dollars is the football team's total payroll than the baseball team's total payroll?

0.46 × 109 dollars
2.8756 × 109 dollars
4.6 × 1011 dollars
2.8756 × 1011 dollars

Answers

Answer: I belive it would be 4.6 x 10^11 dollars. I hope this helps you :)

Step-by-step explanation:

Answer:it would be 4.6x10^11

Step-by-step explanation:

How would the fraction71-√√√5using difference of squares?OA. 7-7√56OB. 7+7√56O c. 7+7√5OD. -7+7√5← PREVIOUSbe rewritten if its denominator is rationalizedSUBMIT

Answers

[tex]D)-\frac{7+7\sqrt{5})}{4}[/tex]

1) Examining that ratio, we can perform the following:

[tex]\begin{gathered} \frac{7}{1-\sqrt{5}} \\ \\ \frac{7\left(1+\sqrt{5}\right)}{\left(1-\sqrt{5}\right)\left(1+\sqrt{5}\right)} \\ \\ \frac{7+7\sqrt{5}}{1^2-(\sqrt{5})^2} \\ \\ \frac{7(1+\sqrt{5})}{-4} \\ \\ -\frac{7(1+\sqrt{5})}{4} \end{gathered}[/tex]

2) Note that when we multiply that ratio by their conjugates, that yields a difference between two squares. Note that on the top, there is the expanded version of this expression.

Thus, the answer is D

translate the following verbal statement into an algebraic equation and solve. Paid 24,998 for a car which was 1,815 less than sticker price what was the sticker price of the caruse x for your vairableequation_______x=______

Answers

paid price = 24,998

it is the amount that is 1815 less than the sticker price,

so the sticker price or price of the car is x

so x = 24,998 + 1815

x =26,813

so the price of car is x = 26,813.

Help on math question precalculus Match the description with the correct base for the logarithm.-LOG without a subscript has a base of -Ln has a base of Choices =10,e

Answers

The formal way of writing a logarithm is the following:

[tex]\log _ab[/tex]

Where "a" is the base of the logarithm and "b2 is the argument.

If "a = 10", then the base is not written, like this:

[tex]\log _{10}b=\log b[/tex]

In the case that the base is the constant number "e" then the logarithm is called a "natural logarithm" and it is written as follows:

[tex]\log _eb=\ln b[/tex]

Other Questions
Describe how your three treatments improved the diffusion rate in your patients lungs. Include concentration difference, diffusion distance, concentration gradient, and surface area and in your answer. Find sin 2x, cos 2x, and tan 2x if tan x= -3/2 and x terminates in quadrant IV. 1) The perimeter of a rectangular garden is 344M. If the width of the garden is 76M, what is its length? Equation:Solution:(I need the equation and solution)2) The area of a rectangular window is 7315CM^2 (^2 is squared). If the length of the window is 95CM, what is its width? Equation:Solution:(Once again, I need the equation and solution)3) The perimeter of a rectangular garden is 5/8 mile. If the width of the garden is 3/16 mile, what is its length?4) The area of a rectangular window is 8256M^2 (^2 is squared). If the length of the window is 86M, what is its width?5) The length of a rectangle is six times its width. The perimeter of the rectangle is 98M, find its length and width. 6) The perimeter of the pentagon below is 58 units. Find VW. Write your answer without variables. I inserted a picture of the question please state whether the answer is a b c or d PLEASE GIVE A VERY VERY SHORT EXPLANATION When considering different hypotheses, usually the _______ one which can account for the ________ is the correct one. Find the indicated quantity, given u = (4, -9), v = (-4, -7).Step 4 of 4: Find (u v)4v. ty received test Graves of 71%, 82%, 71%, 78% and 78%.A) what grade would he need to make on the 6th test to get a C if a C is at least 75% but less than 80%?B) is it possible for tie to get a b or better for his test average at least 80%? Mr. McConnley states that he returned four days ago from a convention in Florida where he both lodged and attended conference meetings at the same hotel. Does this support Dr. Millers suspicions? the helps to explain the situation of massive unemployment, as well as to better understand how an economy will react to such economic shocks as changes in tax rates as well as government and household spending. multiple choice question. investment schedule consumption schedule investment demand curve aggregate expenditure model Labeling the human Skeleton system and labeling nervous system the mean is 5.8the variance is 2.4We have to find the standard deviation and round it to one decimal place. As I am completely brand new to this subject/branch of mathematics, please explain thoroughly, step by step on how to complete this This is a practice from my ACT prep guide take your time, as there is no rush *Ignore the last answer option Calculate the percent by volume of 75 mL of concentrated juice if the juice is diluted with 200 mL of water. how do you write a complete paragraph A total of $6000 is invested: part at 5% and the remainder at 10%. How much is invested at each rate if the annual interest is $590 For circle H, JN = x, NK = 8, LN = 4, and NM = 20.Solve for x. is this equation no solution, one solution, or infinitely may solutions An altitude is drawn from the vertex of an isosceles triangle, forming a right angle and two congruent triangles. As a result, the altitude cuts the base into two equal segments. The length of the altitude is 21 inches, and the length of the base is 18 inches. Find the triangles perimeter. Round to the nearest tenth of an inch. What is a solution of a system of linear equations in three variables? Your mom greets you first thing in the morning. What would she say(The answer needs to be in spanish)